LSAT and Law School Admissions Forum

Get expert LSAT preparation and law school admissions advice from PowerScore Test Preparation.

 ShannonOh22
  • Posts: 70
  • Joined: Aug 15, 2019
|
#71856
Adam Tyson wrote:What makes you think that answer B is the one that is most strongly supported? That is what the stem asks you to find, the answer that is most strongly supported. It isn't about right and wrong. It almost never is about right and wrong on this test.
In regards to the comment above, I have a few questions. How is it that this test is not about right and wrong? Dave and Jon are constantly repeating that it IS in fact about right and wrong - that there are 4 completely WRONG answers, and only ONE that is completely RIGHT. Therefore, even for a question that has "most" in the stem, we are not in fact looking for an answer that is better than the others - we are looking for the only answer that can actually be logically correct. Have I been misinterpreting this message the whole time?!

On another note, I am still a bit confused as to why B is wrong. Here is why I think B is the most strongly supported:

The stimulus first states that scientists solve the problems they are called upon to solve, and that usually these problems are selected by the scientists themselves. This lends itself to a fairly obvious assumption that they would be using a scientific formula to solve these problems.
It then says "When the problems are instead selected by politicians or business leaders, their formulation is nevertheless guided by scientists in such a way as to make scientific solutions feasible."

In context, the "when" here should be taken to mean "anytime" the problems are selected by politicians or business leaders, correct? If so, that means the scientists are using a scientific formula to solve the problems that they themselves select, AND will invariably find a way to solve problems brought to them by politicians and business leaders using a scientific formula, regardless of the nature of the problem itself and whether or not it is suited to a scientific-formula-based solution.

Thus, B) "Any problem a scientist can solve can be formulated in such a way as to make a scientific solution feasible" would follow directly from the stimulus. This statement may not hold water in the real world, but I was also under the impression we are to separate the LSAT stimuli from our conception of the "real world", especially for First Family questions in which we are asked to select an answer based ONLY on information in the stimulus.

I do understand that the word "any" is quite strong...but according to the stimulus, this is in fact true.

Your thoughts/comments/corrections are greatly appreciated!!
User avatar
 KelseyWoods
PowerScore Staff
  • PowerScore Staff
  • Posts: 1079
  • Joined: Jun 26, 2013
|
#72322
Hi Shannon!

While it's true that there are always 4 incorrect answers and only one answer choice that will be credited as correct, we are always looking for the best answer choice out of the 5 options they've given us. From the official LSAT Logical Reasoning instructions: "For some questions, more than one of the choices could conceivably answer the question. However, you are to choose the best answer; that is, the response that most accurately and completely answers the question." We're always making a comparative decision between the answer choice options we've been given. I believe that's what Adam was referring to in his response. He was also trying to get the poster to be more specific about why they thought an answer choice was right. There are usually multiple reasons why an answer choice is correct or incorrect--if we don't know your thinking, it's hard for us to give you a specific explanation that will register with you. So thanks for sharing your thought process for (B)! Let me see if I can clear some things up:

Your thinking goes awry when you make the assumption from the first 2 sentences that scientists would be using a scientific formula to solve the problems they choose. That is actually not an obvious assumption and, in general for MBT and Most Strongly Support questions, you should not be making assumptions. You should be relying on the statements as they are explicitly stated in the stimulus. So even though we know that scientists select problems to solve themselves, we have no explicit evidence to support that that means all the problems they select and solve are able to be formulated in a way that makes a scientific solution feasible. Since we don't know that all of the problems that they select themselves are capable of being solved in a scientific way, we don't know that any problem they solve can be formulated in such a way as to make scientific solutions feasible.

You're absolutely right that you should not be looking for a statement that is true in the real world. It has to be supported by the statements explicitly stated in the stimulus. Just be careful not to make assumptions!

Hope this helps!

Best,
Kelsey

Get the most out of your LSAT Prep Plus subscription.

Analyze and track your performance with our Testing and Analytics Package.